- PowerScore Staff
- Posts: 5972
- Joined: Mar 25, 2011
- Mon Jan 20, 2014 12:00 am
#59725
Complete Question Explanation
(The complete setup for this game can be found here: lsat/viewtopic.php?t=6078)
The correct answer choice is (E)
The question stem creates a KO block. Since no obvious diagramming inferences can be made, reuse information, as was done in question #5.
The hypothetical from Question #1 applies since K and O are next to each other (remember, the first and last variables are next to each other); and this hypothetical proves that L can sit directly between K and M. That is sufficient to eliminate answer choice (A). The work from question #2 also applies and is sufficient to eliminate answer choice (D). Question #3 does not apply. Question #4 does apply and proves that L can sit between M and O (not in the answers) or M and K (already known from question #1). The hypothetical from question #6 must be examined closely. In spite of the fact that M and K rotate in the dual-option, K will always sit next to O, and thus the hypothetical reveals that L can sit between K and N as well as M and N. This information eliminates answer choices (B) and (C). Answer choice (E) is therefore correct via process of elimination.
This question again reveals the power of reusing previous work. With minimal effort all four incorrect answer choices are eliminated. And since the hypotheticals can be visually scanned at a high rate, the question takes less time than a question requiring a new diagram.
Circular Linearity games appear once in a blue moon, and it is not likely that one will appear on your test. Nevertheless, it is worthwhile to be acquainted with the basic principles in case such a game happens to appear.
(The complete setup for this game can be found here: lsat/viewtopic.php?t=6078)
The correct answer choice is (E)
The question stem creates a KO block. Since no obvious diagramming inferences can be made, reuse information, as was done in question #5.
The hypothetical from Question #1 applies since K and O are next to each other (remember, the first and last variables are next to each other); and this hypothetical proves that L can sit directly between K and M. That is sufficient to eliminate answer choice (A). The work from question #2 also applies and is sufficient to eliminate answer choice (D). Question #3 does not apply. Question #4 does apply and proves that L can sit between M and O (not in the answers) or M and K (already known from question #1). The hypothetical from question #6 must be examined closely. In spite of the fact that M and K rotate in the dual-option, K will always sit next to O, and thus the hypothetical reveals that L can sit between K and N as well as M and N. This information eliminates answer choices (B) and (C). Answer choice (E) is therefore correct via process of elimination.
This question again reveals the power of reusing previous work. With minimal effort all four incorrect answer choices are eliminated. And since the hypotheticals can be visually scanned at a high rate, the question takes less time than a question requiring a new diagram.
Circular Linearity games appear once in a blue moon, and it is not likely that one will appear on your test. Nevertheless, it is worthwhile to be acquainted with the basic principles in case such a game happens to appear.
Dave Killoran
PowerScore Test Preparation
Follow me on X/Twitter at http://twitter.com/DaveKilloran
My LSAT Articles: http://blog.powerscore.com/lsat/author/dave-killoran
PowerScore Podcast: http://www.powerscore.com/lsat/podcast/
PowerScore Test Preparation
Follow me on X/Twitter at http://twitter.com/DaveKilloran
My LSAT Articles: http://blog.powerscore.com/lsat/author/dave-killoran
PowerScore Podcast: http://www.powerscore.com/lsat/podcast/